LSAT and Law School Admissions Forum

Get expert LSAT preparation and law school admissions advice from PowerScore Test Preparation.

User avatar
 Dave Killoran
PowerScore Staff
  • PowerScore Staff
  • Posts: 5853
  • Joined: Mar 25, 2011
|
#88162
Complete Question Explanation
(The complete setup for this game can be found here: lsat/viewtopic.php?f=169&p=88159#p88159)

The correct answer choice is (E).

This question asks you for a list of dancers that could be off stage when J is on stage. From the first rule, we know that L must be off stage. Thus, the correct answer must contain L. Answer choice (D) does not contain L, and can be eliminated.

From the third rule, we know that F must be on stage and therefore cannot be off stage. Answer choice (B) contains F, and can thus be eliminated.

From the final rule we know that G must also be on stage, and thus G cannot be included in the correct answer. However, none of the answers includes G.

At this point, we have eliminated answer choices (B) and (D), leaving answer choices (A), (C), and (E) in contention. We have established that J, F, and G are all on stage. As these are the only three dancers that must be on stage, the other three dancers—H, K, and L—can all be off stage. Only answer choice (E) contains all three of these dancers, and thus answer choice (E) is correct.

Get the most out of your LSAT Prep Plus subscription.

Analyze and track your performance with our Testing and Analytics Package.